Sei sulla pagina 1di 56

www.AceMedicine.

com

70 Cardiology, Endocrinology & Rheumatology


SBAs, EMQs & MCQs
CARDIOLOGY EMQ 1
Question: Choose which clinical diagnosis is most likely to be associated with the pulse features
described

a. Atrial fibrillation
b. Aortic stenosis
c. Aortic incompetence
d. Mitral stenosis
e. Supraventricular tachycardia
f. Complete heart block
g. Benign ventricular ectopics
h. Anxiety

Q1: A 52-year-old man, previously fit and well, has become increasingly short of breath on exertion
and has been admitted following a dizzy spell. Amongst other clinical signs you notice that his pulse
is 60 beats per minute, sinus rhythm. It is slow rising.

A1: B. Aortic Stenosis (AS). In AS the pulse is typically slow and/or has a sustained upstroke of the
arterial pulse, and the pulse may be of low volume. This is because the left ventricle is working hard
to drive blood out thorough the narrowed aortic valve. There may also be a noticeable delay
between the first heart sound (on auscultation) and the corresponding pulse in the carotid artery
(so-called 'apical-carotid delay'). Similarly, there may be a delay between the appearance of each
pulse in the brachial artery (in the arm) and the radial artery (in the wrist).

Q2: A 46-year-old man attends for outpatient assessment following referred by his GP. He
complains of an episodic sudden onset of very a fast pulse rate, and a fluttering sensation within his
chest. He takes his pulse at the time and states the rate can be greater than 200 beats per minute.
Each episode lasts typically for few minutes and then reverts spontaneously. His GP had requested a
24 hour tape recording, but this was normal.

A2: E. Supraventricular Tachycardia (SVT). This is the classical history associated with SVT. It often
presents in younger patients, although it can be seen in older patients. The arrhythmia typically
arises from the atria and therefore the rate is often greater than 200 beats per minute. Episodes are
often self-terminating, but if treatment is required adenosine is usually the first line drug. Beta-
blockers are also useful. If haemodynamically unstable cardioversion should be performed as it is
safe and effective.
www.AceMedicine.com

Q3: A 46-year-old Asian gentleman attends the outpatient department referred by his GP. He
complains of increasing shortness of breath and a reduced exercise tolerance at the gym. He has
been fit and well all his life. Amongst other clinical signs the pulse has a tall upstroke and then
seems to ‘fall away’. Blood pressure is 140/50.

A3: C-Aortic incompetence. A collapsing pulse and wide pulse pressure are almost always diagnostic
of some degree of aortic incompetence. It is associated with a congenital bicuspid aortic valve and
symptoms may begin on in the 3rd or 4th decade of life. SOB is usually the predominant symptom.
Echocardiography is the most useful investigation, helping to quantify the degree of incompetence
and plan treatment. If there is evidence of left ventricular dilatation then ACE inhibitors will be
prescribed.

Q4: An 84 year old farmer is admitted to the acute medical receiving unit with dizzy spells. His pulse
is very slow and irregular. His blood pressure is 120/80 mmHg. There are intermittent irregular
visible pulsations noted on inspection of his venous pressure

A4: F. Complete Heart Block. Patients with complete AV block typically experience bradycardia (as
low as 28 beats per minute during sleep), hypotension, and at times, hemodynamic instability. In
some cases, exercising may be difficult, as the heart cannot react quickly enough to sudden changes
in demand or sustain the higher heart rates required for sustained activity. They often present with
dizzy spells. In this case treatment will be referral for a permanent pacemaker, however a temporary
wire is not required as he is haemodynamically stable.

Q5: A 46 year old consultant endocrinologist at your hospital has been complaining of irregular
episodes of palpitations for a few years. They are worse following ingestion of alcohol and coffee
and also tend to be worse at night. On examination heart sounds are normal. The pulse is irregular at
times. Echocardiography is entirely normal.

A5: G. Benign Ventricular Ectopics. This is a benign condition caused by ventricular extra-systoles and
tends to have a benign course. A typical history is feeling an extra-beat followed by a slight pause
and then the heart beat returns to normal. Usually patients have a structurally normal heart and 24
hr tape simply reveals ectopic beats.
www.AceMedicine.com

CARDIOLOGY EMQ 2
Question: Choose which clinical diagnosis is most likely to be associated with the murmurs described

a. Ventricular septal defect


b. Aortic stenosis
c. Aortic incompetence
d. Mitral stenosis
e. Pulmonary stenosis
f. Mitral incompetence
g. Tetralogy of fallot
h. Tricuspid incompetence

Q1: A 26-year-old patient with Noonan Syndrome. On auscultation you hear a soft ejection systolic
murmur at the left upper sternal border, louder with inspiration.

A1: E. Pulmonary Stenosis. The classical murmur is heard at the left upper sternal border and is
louder with taking a deep breath (which increases venous return). Pulmonary stenosis is associated
with Noonan syndrome, a relatively common genetic condition, and a type of dwarfism. It affects
males and females equally and about half have pulmonary stenosis.

Q2: A 46-year-old Asian gentleman with blood pressure 140/50 mmHg. You hear an early diastolic
murmur over the upper sternal border louder with expiration.

A2: C. Aortic Incompetence. Early diastolic murmurs often point to outflow tract valvular
regurgitation. The fact that it is louder with breathing out suggests a murmur on the left side of the
heart
Q3: A 22-year-old patient with Downs’ syndrome. There is evidence of finger clubbing and resting
oxygen saturations are 87%. On auscultation you hear a loud 4/6 pan-systolic murmur at the lower
sternal border.

A3: A. VSD with subsequent development of Eisenmenger syndrome. The diagnosis is ventricular
septal defect (VSD). The patient has trisomy 21 which has a clear association with cardiac
malformations including VSD. Eisenmenger syndrome develops whenever there is a persistent
uncorrected left to right cardiac shunt, which over time leads to chronically elevated right sided
pressures and ultimately pulmonary hypertension. Patients become cyanotic and prognosis is poor.

Q4: A 64 year old gentleman 4 days following primary PCI for an LAD infarct. He becomes short of
breath and hypotensive. On examination there is a loud pansystolic murmur at the lower sternal
border and apex. Echocardiogram reveals turbulent flow in the region of the septum on para-sternal
long-axis view.
www.AceMedicine.com

A4: A. VSD. This is an acquired VSD, which may be a consequence of a large infarct. It is rare, but may
happen and clinicians should be aware of the possibility as prompt treatment is required. Usually a
discussion with the cardiac surgeons will be needed.

Q5: A 67-year-old lady has become progressively short of breath. You hear a loud ejection systolic
murmur which radiates to both carotids.

A5: B. Aortic Stenosis. Aortic stenosis is a common problem. Approximately 2% of people over the
age of 65, 3% of people over age 75, and 4% percent of people over age 85 have the disorder.
Treatment is usually by conventional surgery and the cut-off is either a gradient greater than
60mmHg, or the development of symptoms. In those not fit for surgery there is increasing use of
newer percutaneous methods.
www.AceMedicine.com

CARDIOLOGY EMQ 3
Question: Which of the following JVP appearances is most likely to be seen in the clinical pictures
described?

a. Raised JVP, normal waveform


b. JVP rising with inspiration
c. Large ‘a’ wave
d. Absent ‘a’ wave
e. Large ‘v’ wave
f. Cannon ‘a’ wave
g. Elevated JVP but no waveform, or change with breathing
h. Slow ‘y’ descent

Q1: An 84-year-old lady with moderate impairment of left ventricular ejection fraction. She has
recently undergone hip replacement and has a positive 4 litre fluid balance in the last 2 days.

A1: A. Raised JVP, normal waveform. This is most likely simple fluid overload and in the presence of
reasonable renal function will respond to appropriate therapy with IV diuretics. The JVP will be
elevated as the right side of the heart fails to clear the backload of fluid returning form the venous
system.

Q2: An 84 year old farmer is admitted to the acute medical receiving unit with collapse. His pulse is
very slow and irregular. His blood pressure is 62 mmHg systolic and he is sweaty. There are
intermittent irregular visible pulsations noted on inspection of his venous pressure.

A2: F. Cannon ‘a’ wave. Patients with complete AV block typically experience bradycardia (as low as
28 beats per minute during sleep), hypotension, and as in this case, hemodynamic instability. In this
case the cannon ’a’ wave is caused by the lack of synchronization between the atria and the
ventricles. The ventricles occasionally contract against a closed AV valve, giving rise to the JVP
appearance. A temporary wire is required as he is haemodynamically unstable.

Q3: A 45-year-old lady is an inpatient in the oncology department with metastatic melanoma. She
has received a recent course of chemotherapy, but has developed acute shortness of breath and is
hypotensive. Bedside echocardiogram reveals a 2.5cm circumferential pericardial effusion.

A3: B. JVP rising with inspiration. This is called Kassmaul’s sign, or “paradoxical” rise in the JVP with
inspiration. Normally the venous pressure (and the JVP) falls with a deep breath as intrathoracic
venous return rises (this is why murmurs on the right side of the heart are louder with inspiration). If
the JVP rises with inspiration it is ‘paradoxical’ and signifies that an extrinsic force, in this case, an
effusion, compresses the ventricle. Treatment is peri-cardiocentesis. Effusions are very common in
malignant disorders, but easily treatable.
www.AceMedicine.com

Q4: A 65-year-old gentleman has recently undergone radiotherapy treatment for lung carcinoma.
He is progressively short of breath and CT scan of the chest demonstrates mediastinal
lymphadenopathy and dilated neck veins. A diagnosis of SVC obstruction is made.
A4: G. Elevated JVP, but no waveform, or change with breathing. Patients with SVC obstruction have
a persistently elevated JVP. It is usually associated with tumor invasion although may also result
from the use of vascular devices and procedures e.g. neck lines. Treatment is usually in the form of
steroids if it is caused by malignancy, or diuretics.

Q5: A 54 year old lady has recently undergone pacemaker lead extraction for endocarditis. It was a
difficult procedure and the ventricular lead was tethered to the tricuspid valve. Echo now confirms
severe tricuspid incompetence.

A5: F. Large ‘v’ wave. A large ‘v’ wave typically suggests tricuspid incompetence. In this case the
tricuspid valve has been damaged during the lead extraction and is regurgitant. The JVP will
demonstrate large ‘v’ waves during ventricular systole. The liver may be enlarged and is often
pulsatile (the latter finding being virtually diagnostic of tricuspid insufficiency). Peripheral edema is
often found. In severe cases, there may be ascites.
www.AceMedicine.com

CARDIOLOGY EMQ 4
Question: what is the most likely diagnosis from the ECG description provided?

a. Acute inferior myocardial infarction


b. Atrial fibrillation
c. Pericarditis
d. Anterior myocardial infarction
e. Complete AV block
f. Right Ventricular Hypertrophy
g. Ventricular Tachycardia
h. Supraventricular Tachycardia

Q1: A 54-year-old man with a history of smoking presents with sudden onset of cheat pain and ST
elevation across leads V1-V4.

A1: D. Anterior myocardial infarction. This is an acute anterior (probably LAD) infarct and the patient
should receive primary PCI if available. If PCI is not available, and there are no contraindications, he
should receive thrombolytic therapy. The most likely vessel to be occluded is the left anterior
descending artery. The aim of treatment is to re-establish flow in the vessel.

Q2: A 26-year-old gentleman complains of chest pain for two days. He had a cold last week and the
pain is worse when he takes a deep breath. The ECG demonstrates global saddle-shaped ST
elevation.

A2: C. Pericarditis. Pericarditis typically causes ST elevation across all leads on the ECG. It is also
described as ‘saddle’ shaped. It tends to resolve with anti-inflammatory therapy and an
echocardiogram should be performed to rule out any effusion. It is often preceded by an influenza
type illness.

Q3: A 33-year-old lady with Tetalogy of Fallot (TOF) attends for annual review. She had corrective
surgery as a young child, with closure of the VSD and placement of a pulmonary homograft. She
keeps well. On inspection of her ECG there are tall R waves in leads V1-V4 and a deep s wave in lead
I.

A3: F. Right Ventricular Hypertrophy. Patients with TOF almost always have RVH (Tetralogy of Fallot
corresponds to 1. VSD; 2. Over-riding aorta; 3. Pulmonary stenosis; and 4. RVH).

Q4: A 13-year-old girl attends for outpatient assessment referred by GP. She complains of episodic
sudden, immediate onset of very fast pulse rate, and a fluttering sensation within her chest. Her
mother takes her pulse at the time and states the rate can be greater than 200 beats per minute.
The 24 hour tape has been performed and demonstrates 2 episodes of fast heart rate. It looks
regular. The rate is 190 per minute and the QRS duration is 90ms.
www.AceMedicine.com

A4: H. Supraventricular Tachycardia. SVT arises from the atria. If there is aberrant conduction it may
mimic a wide-complex tachycardia. Symptoms can come on suddenly and may go away without
treatment. They can last a few minutes or as long as 1 or 2 days, sometimes continuing until treated.
The rapid beating of the heart during SVT can make the heart a less effective pump so that the
cardiac output is decreased and the blood pressure drops. If a patient is hypotensive they should
undergo electrical cardioversion which is safe and effective
Q5: A 64 year old lady with a history of rheumatic heart disease attends out-patients. She is thin, has
a malar flush and has had a St Judes mitral valve replacement 3 years previously. She is now very
short of breath when before this wasn’t a problem. ECG does not show any discernable p wave and
the tracing has a ‘saw-tooth’ appearance. The QRS complex duration is 160ms. There is a negative
deflection in lead V1.

A5: B. Atrial fibrillation. Almost all patients with rheumatic mitral valvular disease develop AF at
some point. The saw tooth nature of this ECG would suggest flutter although it is hard to be
definitive until you see the ECG. The negative deflection in lead V1 also suggests left bundle branch
block, which is very common following surgery.
Chris Lockhart
www.AceMedicine.com

CARDIOLOGY EMQ 5
Question: for each of the scenarios involving cardiac chest pain, what is the best treatment strategy?

a. Dual anti-platelet therapy


b. Dual anti-platelet therapy, low molecular heparin
c. Warfarin
d. Thrombolytic therapy
e. Long acting nitrate preparation
f. Non-steroidal anti-inflammatory preparations
g. Primary percutaneous coronary intervention
h. GTN spray

Q1: A 56-year-old gentleman presents to the emergency room in a large teaching hospital with
central crushing chest pain and ST elevation in leads II, III, and aVF.

A1: G. Primary percutaneous coronary intervention. There is now convincing evidence that the best
treatment for acute STEMI is primary PCI. Of course the patients will have been loaded with aspirin
and plavix first, and will receive unfractionated heparin during the procedure. However, the best
outcomes are seen in those who get immediate access to percutaneous revascularisation and
stenting.

Q2: A 26-year-old gentleman complains of chest pain for two days. He had a cold last week and the
pain is worse when he takes a deep breath. The ECG demonstrates global saddle-shaped ST
elevation.

A2: F. Non-steroidal anti-inflammatory preparations. The diagnosis is pericarditis and treatment is


usually conservative with NSAIDs. Troponin rises are often seen and in the presence of ST elevation ,
doctors are often tempted to treat as acute coronary event. However, with careful clinical
examination and scrutiny of the ECG the diagnosis can usually be made correctly.

Q3: A 72-year-old lady underwent CABG 4 years ago. She has now started complaining of chest
discomfort typically in the morning when she goes out to walk her dog. It is worse when going up
hills. She is already on aspirin, a statin and a beta blocker.

A3: E. Long acting nitrate. This is stable angina. The options here are e. or h. GTN spray may be
considered, but probably will often have a very temporary effect and probably a better option is a
longer acting preparation with sustained release throughout the day.

Q4: A 64-year-old man has just been discharged following percutaneous coronary intervention. He
was treated with drug-eluting stents. What combination of drugs must he remain on for at least a
year?
www.AceMedicine.com

A4: A. Aspirin and plavix (dual anti-platelet therapy). Following the use of drug-eluting stents there is
now clear evidence from a number of trials that patients should remain on both aspirin and plavix
for one year, and then to switch to aspirin alone. This is because of the risk of sub acute stent
thrombosis. Typically there is a 1% risk of acute stent thrombosis in the first 6 weeks. There is about
a 10% risk of instent re-stenosis over the next one year.
Q5: A 24 year old lady has just been discharged following successful percutaneous closure of an ASD
with an amplatzer device. She complains of palpitations and ECG confirms AF. Subsequent 24 hour
tape confirms two very short-lived episodes of AF.

A5: A. Aspirin and plavix. The AF is short-lived and tends to happen after a device is placed in the
atrial septum. In this case aspirin and plavix are used for 6 moths until the device has endothelialised
and then stopped. If the AF was permanent then warfarin and DCC should be considered.
www.AceMedicine.com

CARDIOLOGY EMQ 6
Question: For each of the arrythmias described what would be your first line treatment strategy?

a. Lignocaine
b. Cardioversion
c. Adenosine
d. Amiodarone
e. Digoxin
f. Metoprolol
g. Warfarin
h. Magnesium sulphate

Q1: A 56-year-old gentleman has recently recovered from a large anterolateral infarct. He presents
to A&E with heaviness in his chest and is clod and clammy. ECG demonstrates a wide-complex
tachycardia. There is concordance across all leads. The rate is 170 bpm. Blood pressure is maintained
at 120/80 mmHg.

A1: A. Lignocaine. This is Ventricular Tachycardia (VT). Because the blood pressure is maintained
lignocaine is a good first line agent. It is a membrane stabiliser and is used effectively for VT. If the
patient were to become haemodynamically unstable he should be cardioverted.

Q2: A 72-year-old lady has just undergone a laparoscopy for acute cholecystitis. She is pyrexic,
although white cell count and CRP is normal. She has developed atrial fibrillation.

A2: D. Amiodarone. This is post-operative AF and chemical cardioversion with amiodarone is a


reasonable option. She almost certainly is on IV antibiotics, as infection may be the precipitating
factor causing the AF, as opposed to any underlying cardiac abnormality.

Q3: A 17-year-old girl attends assessment referred by GP. She complains of episodic sudden,
immediate onset of very fast pulse rate, and a fluttering sensation within her chest. You take her
pulse and it is greater than 200 beats per minute.

A3: C. Adenosine. Adenosine causes transient complete AV block, causing asystole and therefore
breaking the cycle of re-entrant tachycardia. It is therefore useful in the treatment of SVT, but is
associated with chest discomfort and patients often feel very unwell. It is, however, very short-
acting and thus its effects do not last for long.

Q4: A 46-year-old man attends for assessment referred by his GP. He complains of episodic sudden,
immediate onset of very fast pulse rate, and a fluttering sensation within his chest. The pulse rate is
greater than 200 bpm. ECG demonstrates a wide-complex tachycardia and the blood pressure is
60mmHg systolic
www.AceMedicine.com

A4: B. Cardioversion. It is hard to speculate as to the exact rhythm although it is probably VT. It may
also be SVT with aberrant conduction. In any case given the haemodynamic compromise this patient
should be sedated, and cardioverted without delay. This is the safest option in this case.

Q5: A 22-year-old lady has just undergone a laparotomy for acute bowel perforation. She is pyrexic
with raised white cell count and CRP. She has developed atrial fibrillation with fast ventricular
response. You note she has a history of rheumatoid arthritis with lung involvement and PFTS suggest
a restrictive pattern

A5: E. Digoxin. This is post-operative AF but in this case amiodarone should be avoided. She is
relatively young and has evidence of pulmonary fibrosis. Therefore amiodarone would be
contraindicated. Rate control with digoxin is a good option and infection should be treated with IV
antibiotics.
www.AceMedicine.com

CARDIOLOGY EMQ 7
Question: For each of the clinical scenarios and corresponding congenital cardiac anatomical defects
described below, what is the correct term?

a. Tetralogy of fallot
b. Congenital correction of transposition of the great arteries
c. Atrioventricular septal defect
d. Ebsteins anomaly of the tricuspid valve
e. Pulmonary stenosis
f. Ventircular septal defect
g. Atrial Septal defect
h. Patent Foramen Ovale

Q1: A 56-year-old university lecturer has been complaining of migraine for two years. He is referred
to the medical outpatient dept with an episode of left arm weakness that spontaneously resolved.
CT brain and echocardiogram are normal. A bubble contrast echo, however clearly demonstrated
bubbles passing from the right atrium into the left atrium, increased following the valsalva
manoeuvre. What is the diagnosis?

A1: H-Patent foramen ovale. PFO is very common (25% of the population). It has been causally linked
with migraine; the mechanism thought to be small emboli passing from the defect to the left sided
systemic circulation and thus causing micro-infarcts. When it is associated with larger emboli, say
from a DVT, and giving rise to TIA or indeed CVA, the term cryptogenic stroke is used. Common
convention is to close the PFO with an occluder device from the venous system if there have been
neurogical sequelae. However, clinical trials are still awaited and other clinicians feel warfarin is
better for prevention of events.

Q2: A 21-year-old girl with trisomy 21 and arterial saturations of 81% at rest. Echocardiogram shows
bidirectional flow at the upper end of the septum involving the mitral valve, which is also leaking.
What is the diagnosis?

A2: C-Atrioventricular septal defect. The anatomy described is AVSD, which is commonly seen in T21.
This girl unfortunately has not had this defect repaired and thus has developed Eisenmenger
complex. This arises when there is sustained left to right flow for many years which leads to
chronically high right sided pressures.

Q3: A 41-year-old man with O2 saturations of 78% on room air and is increasingly SOB. There is a
loud pan-systolic murmur and a pulmonary ejection systolic murmur. Echocardiogram demonstrates
the left ventricle is anterior and supplies the pulmonary circulation. There is also a VSD. What is the
diagnosis?

A3: B-Congenital correction of transposition of the great arteries (CCTGA). Transposition of the great
vessels means the aorta arises from the RV and the pulmonary arteries arise from the LV. It is not
www.AceMedicine.com

compatible with life unless there is a co-existent septal defect to allow blood to mix. In CCTGA the
ventricles are anatomically switched. There is an anterior LV which gives rise to the pulmonary
circulation and the RV is the systemic ventricle. It is often associated with pulmonary stenosis and
VSD.

Q4: A 2 week old neonate with failure to thrive. Echo reveals a VSD, over-riding aorta, pulmonary
stenosis and a RV impairment. What is the diagnosis?
A4: A-Tetralogy of Fallot. This combination is tetralogy of fallot. Children usually do well with
corrective surgery but will often require a pulmonary valve replacement (homograft) later in life.

Q5: A 6-year-old child with Noonan syndrome is seen at the developmental clinic. There is a harsh
ejection systolic murmur louder on deep inspiration. Echo shows turbulent flow in the right
ventricular outflow tract.

A5: E-Pulmonary Stenosis. Noonan syndrome is associated with pulmonary stenosis. Right sided
murmurs are louder on inspiration as a deep breath increases systemic venous return ie to the right
side of the heart and thus the murmur intensity is increased.
www.AceMedicine.com

CARDIOLOGY EMQ 8
Question: For each clinical scenario described what is the lost likely cause of the hypertension.

a. Cushings syndrome
b. Acromegaly
c. Renal artery stenosis
d. Phaeochromcytoma
e. Coarctation of aorta
f. Type 1 diabetes
g. Essential hypertension
h. Sub-dural haematoma

Q1: A 76-year-old gentleman is admitted with increasing subtle confusion. Clinical examination is
normal, with no lateralising neurological signs. You notice however that his BP is 182/90 mmHg, and
his pulse is 42/min. Blood pressure has always been normal beforehand according to the GP. There
is a remote history of a fall 5 weeks ago. What is the diagnosis and most likely cause of
hypertension?

A1: H. Sub-dural haematoma. This is the most likely diagnosis here given the clinical details.
Cushing’s reflex denotes that in order to maintain cerebral perfusion in the setting of raised
intracranial pressure, blood pressure rises and heart rate often falls.

Q2: A 46-year-old lady with increasing obesity, and a new diagnosis of osteoporosis and type 2
diabetes. Blood pressure is 156/90 mmHg. What is the unifying diagnosis?

A2: A-Cushing’s syndrome. This is the most likely diagnosis. It may arise secondary to steroid use,
pituitary disease or secondary to exogenous ACTH or cortisol production. It can be diagnosed
through an overnight dexamethasone suppression test.

Q3: A 35-year-old man with intermittent flushing and palpitations. 24-hour blood pressure monitor
reveals average daytime pressure of 178/96mmHg. What is the most likely diagnosis?

A3: D-Phaeochromocytoma. Although a rare condition, given the symptoms, phaeochromocytoma


needs to be excluded, as it is readily treatable.

Q4: A 60-year-old lady presents with acute shortness of breath. Her oxygen level (PaO2) is low at
6.1. CXR reveals unilateral pulmonary oedema and there is left bundle branch block on the ECG.
Coronary angiography had been performed 2 months previously and was known to be normal. BP is
210/110 mmHg. What is the most likely diagnosis?

A4: C-Renal artery stenosis. This is a rare cause of hypertension and can rarely give rise to unilateral
pulmonary oedema. The exact mechanism for this is not known. Diagnosis is by ultrasound or
magnetic resonance imaging of the renal vessels.
www.AceMedicine.com

Q5: A 26 year old girl presents 18 weeks pregnant. She has been increasingly SOB. It emerges she
was told she had a heart murmur as a child. You examine her and note a systolic murmur radiating
through to the back and the femoral pulses are very difficult to feel. What is the diagnosis?
A5: E. Coarctation of aorta- mild coarctation may often be asymptomatic. However, the weak distal
pulses suggest this as a diagnosis. She will need an echocardiogram and will require surgical
correction following delivery.

CARDIOLOGY EMQ 9

The haemodynamically unstable patient

a. Pulmonary embolism
b. Inferior myocardial infarction
c. Cardiac tamponade
d. Left ventricular failure
e. Aortic dissection
f. Severe mitral regurgitation
g. Severe aortic stenosis
h. Right ventricular infarction
i. Hypertrophic cardiomyopathy
j. Coarctation of the aorta

1. A 66 year old man presents with a 2 day history of back/interscapular pain which is sharp and
tearing in nature. He is a known hypertensive, but BP on admission is 85/40. His 12 lead ECG shows
evidence of ST elevation in leads II,III and aVF. On examination he has evidence of severe aortic
regurgitation, not previously documented, with high volume pulses.

e) This patient gives a suitable history and features for acute aortic dissection, although on
oaccasions the history may be more of acute onset (minutes/hours) of intrascapular/chest pain. If
the dissection flaps extends to include the ascending aorta/aortic root then the right coronary artery
can become compromised resulting in an inferior myocardial infarction. However, this patient
should not be thrombolysed until a dissection has been excluded with a contrast enhanced CT of the
thorax. Involvement of the aortic root can also compromise the aortic valve resulting in aortic
regurgitation

2. A 72 year-old man becomes hypotensive 2 hours after apparently successful thrombolysis for an
inferior myocardial infarction. A right praecordial lead ECG shows normal ST segments in V4R to V6R.
On examination there is a new systolic murmur and the LV apex is thrusting. There is florid
pulmonary oedema.

f). Mechanical complications of myocardial infarction include papillary muscle infarction (resulting in
severe mitral regurgitation) and VSD which can both cause hypotension. The findings of a normal RV
www.AceMedicine.com

lead ECG, a systolic murmur and volume overloaded apex make acute MR the most likely diagnosis
although an echocardiogram would need to be performed to exclude the formation of a VSD which
tends to occur with anterior myocardial infarctions. Acute MR results in severe pulmonary oedema.

3. A 63 year old lady is diagnosed as having a posterior myocardial infarction, for which she is
thrombolysed. She becomes progressively more hypotensive, and is observed to have an elevated
JVP. Her lung fields are clear and auscultation is normal.

h) Right ventricular infarction is the most commonly associated with an inferior MI, but can also be a
complication of a posterior circulation infarct, particularly in patients who have a dominant
circumflex system. In patients suffering a posterior MI who become hypotensive, a right precordial
lead ECG should be performed.

4. A 33 year old lady presents with a syncopal episode 1 week after ankle surgery. Her BP is 75/34,
her heart rate is 160 bpm in atrial fibrillation and her PaO2 is 10.5 kPa on 60% FiO2. Her chest x-ray
is clear.

a) A pulmonary embolus should be suspected in any patient who is hypoxic and tachycardic,
particularly if there is evidence of new atrial fibrillation/flutter. The marked hypotension in this case
is a worrying sign and suggests the patient may be suffering a massive PE, with the embolus
obstructing both pulmonary arteries at the bifurcation of the main pulmonary artery (saddle
embolus) and she requires an urgent CT pulmonary angiogram and consideration for thrombolysis.

5. An 86 year-old man presents with a 2 week history of worsening dyspnoea and syncope. He has a
weak pulse and his BP is 110/85. On auscultation there is a soft cresendo-decresendo systolic
murmur. The first heart sound is present, but S2 is absent. There are crackles throughout both lungs
and CXR confirms pulmonary oedema.

g). This patient has features of severe aortic stenosis, such as low volume pulse, narrow pulse
pressure, an ejection systolic murmur and absent S2. A soft or inaudible S2 suggests that the aortic
valve is severely calcified and has lost its pliability, resulting in a loss of the closing sound of aortic
valve closure. Severe aortic stenosis manifests with chest pain (angina), dyspnoea (increased left
atrial pressure) and syncope (overstimulation of mechanoreceptors in the left ventricle triggering a
reflex bradycardia/vasodilatory response)
www.AceMedicine.com

CARDIOLOGY SBA 1
A 32-year-old gentleman presents to the emergency department with pleuritic chest pain, mild SOB
and pyrexia. The SHO in A & E is concerned about the ECG appearance and contacts you. It
demonstrates saddle shaped ST elevation across all leads.
What is the appropriate therapeutic option in the first instance pending further investigation?

A. Beta Bocker
B. Aspirin
C. NSAID
D. Reteplase
E. Low molecular weight heparin

c. NSAID
The clinical history suggests pericarditis and this is certainly the most likely diagnosis given the initial
ECG. If symptoms improve following NSAID use then this also points towards the potential diagnosis.
He should also have an echocardiogram and possibly a CTPA is clinically merited.

CARDIOLOGY SBA 2
A 64-year-old gentleman presents with markedly increased SOB on exertion and feeling generally
fatigued all the time. This has occurred gradually over the last few months. He also complains of a
fast pulse that is irregular. ECG confirms atrial fibrillation with a fast ventricular response;
furthermore he is hypotensive. Echocardiogram is normal. You elect for rate control with DCC as an
inpatient.
Which investigation is mandatory before cardioversion ?

A. CXR
B. INR
C. CT pulmonary angiogram
D. Coagulation screen
E. Trans-oesophageal echocardiogram (TOE)

e. TOE
As you have elected for cardioversion and his symptoms have lasted greater than 24 hours then
there is a theoretical risk of LA appendage thrombus. This is a contraindication to electrical
cardioversion and therefore TOE needs to be performed in this case to rule this out.

CARDIOLOGY SBA 3
A patient presents with heart failure following a number of myocardial infarcts. Their symptoms are
difficult to control and you prescribe carvedilol, furosemide, perindopril, spironolactne and
metolozone. They are also on warfarin therapy as the ejection fraction is less than 20% on echo.
Following discharge they present with profound hypokalaemia.
Which drug would you withdraw first?
www.AceMedicine.com

A. carvedilol
B. furosemide
C. perindopril
D. spironolactone
E. metolozone

e. metolozone.
Metolozone is a potent thiazide diuretic and thus can cause profound hypokalaemia. Furosemide
can also cause low potassium but is not as strong. Carvedilol, perindopril and spironolactone all have
a tendency to raise serum potassium and should be continued.

CARDIOLOGY SBA 4
A 47-year-old lady with a background history of scleroderma presents with progressive shortness of
breath. Her lungs are clear but there is a pan-systolic murmur at the lower left sternal border. CXR
and CT chest are normal. Echocardiogram reveals severe tricuspid incompetence and an estimated
pulmonary artery pressure of 89mmHg.
Which therapeutic modality would be the first line agent of choice in this case?

A. sildenafil
B. carvedilol
C. salbutamol
D. bosentan
E. inhaled prostacyclin

a. sildenafil.
This is primary pulmonary hypertension, a progressive and serious condition which often requires
treatment. It is related to autoimmune rheumatological disorders. In this case first line treament
would be sildenafil which promotes pulmonary vasodilatation and thus reduces PA pressure. If this
fails then bosentan (an endothelin receptor antagonist) and prostacyclin may be of benefit.

CARDIOLOGY SBA 5
A 65-year-old gentleman with a history of myocardial infarction presents feeling unwell and dizzy. He
is cold, clammy and his pulse is racing. There is no chest pain. ECG shows a wide-complex
tachycardia, rate 200/minute, with concordance across all leads. Blood pressure is 72mmHg systolic.
What is the optimal treament?
A. IV metoprolol
B. IV amioderone
C. DC cardioversion (DCC)
D. IV lignocaine
E. Verapamil

c. DCC
www.AceMedicine.com

This is VT until proven otherwise and should be treated as such. In the presence of hypotension the
safest option is DCC, with sedation beforehand. If there is no hypotension then lignocaine is a good
option.

CARDIOLOGY SBA 6
A 54-year-old woman has just returned to the ward following successful ablation for a
supraventricular tachycardia. She begins to feel unwell and is found to be SOB, tachycardic and
hypotensive, with pulsus paradoxus of 20mmHg. She remains in sinus rhythm, but her JVP is raised
and does not fall with inspiration.
What initial treatment is indicated?

A. Repeat ablation
B. Coronary angiography
C. CPAP
D. Pericardiocentesis
E. IV beta blocker

d. Pericardiocentesis
Cardiac Tamponade is a complication of ablation occurring in about 1% of cases. It is readily
treatable by pericardiocentesis if performed promptly. It is confirmed by echocardiography and it
only takes 100mls of blood to cause tamponade if it collects rapidly.

CARDIOLOGY SBA 7
A 62-year-old gentleman with a history of surgical correction of a aortic coarctation in childhood,
presents at the clinic progressively SOB. There is an ejection systolic murmur heard best over the
aortic area. You diagnose aortic stenosis.
What is the most likely aetiology of the aortic stenosis?

A. Aortic sclerosis
B. Age degeneration
C. Post surgery to correct coarctation
D. Bicuspid aortic valve
E. Post infective endocarditis

D. Bicuspid aortic valve


Bicuspid aortic valve is often associated with coarctation and often leads to aortic stenosis requiring
surgery later in life.
www.AceMedicine.com

CARDIOLOGY SBA 8
An 84-year-old farmer is admitted to the acute medical receiving unit with dizzy spells. His pulse is
very slow and irregular. His blood pressure is 120/80 mmHg. There are intermittent irregular visible
pulsations noted on inspection of his venous pressure. ECG confirms rate of 35/min.
What is the most likely diagnosis?
A. Slow AF
B. Aortic stenosis
C. Complete Heart Block
D. Sinus Bradycardia
E. Postural hypotension
c. Complete Heart Block
The history is very suggestive of CHB. Whilst haemodynamocally stable the pulse and JVP features
strongly suggest complete AV dissociation.

CARDIOLOGY SBA 9
A 46-year-old man attends for outpatient assessment referred by GP. He complains of episodic
sudden, immediate onset of very fast pulse rate, and a fluttering sensation within his chest. He takes
his pulse at the time and states the rate can be greater than 200 beats per minute. Each episode
lasts typically for few minutes and then reverts spontaneously. His GP had requested a 24 hour tape
recording, but this was normal. ECG demonstrates a delta upstroke and tall R waves in leads V1-V3
indicative of pre-excitation.
What is the most likely diagnosis?
A. Ventricular Tacyhcardia
B. Sinus Tachycardia
C. Anxiety Disorder
D. Wolf Parkinson White
E. Phaeochromocytoma

d. Wolf Parkinson White


This is the classical history associated with SVT. Wolf Parkinson White is a form of SVT denoted by
evidence of pre-excitation on the ECG. It can often present in younger patients, although also seen in
older patients. The arrhythmia typically arises from the atria and therefore the rate is often greater
than 200 beats per minute. Episodes are often self-terminating, but if treatment is required
adenosine is usually the first line drug. Beta-blockers are also useful. You should not use digoxin or
verapamil as it can promote excitation through the accessory pathway. If haemodynamically
unstable cardioversion should be performed as it is safe and effective.

CARDIOLOGY SBA 10
A 56-year-old university lecturer has been complaining of migraine for two years. He is referred to
the medical outpatient department with an episode of left arm weakness that spontaneously
resolved. CT brain and echocardiogram are normal. A bubble contrast echo, however clearly
demonstrated bubbles passing from the right atrium into the left atrium, increased following the
valsalva manoeuvre.
www.AceMedicine.com

What is the diagnosis?


A. Patent Foramen Ovale
B. Atrial septal Defect
C. Atrioventricular Septal defect
D. VSD
E. Normal physiology

a. Patent Foramen Ovale.


PFO is very common (25% of the population). It has been causally linked with migraine; the
mechanism thought to be small emboli passing from the defect to the left sided systemic circulation
and thus causing micro-infarcts. When it is associated with larger emboli, say from a DVT, and giving
rise to TIA or indeed CVA, the term cryptogenic stroke is used. Common convention is to close the
PFO with an occluder device from the venous system if there have been neurogical sequelae.
However clinical trials are still awaited and other clinicians feel warfarin is better for prevention of
events.

Cardiology SBA 11

In the investigation of infective endocarditis, which of the following statements are correct?
a) According to the Duke criteria, auscultation of a new murmur fulfils a minor criterion for the
diagnosis of infective endocarditis
b) The presence of haematuria is a common finding in endocarditis
c) The presence of a vegetation can usually be detected with transthoracic echocardiography
d) Elevated levels of CRP and erythrocyte sedimentation rate (ESR) fulfil 1 minor criterion of the
Duke criteria for diagnosing infective endocarditis.
e) A normal transoesophageal echocardiogram rules out the diagnosis of infective endocarditis

b). The presence of haematuria is a common finding in endocarditis

The clinical diagnosis of infective endocarditis is made using the criteria developed by a group from
Duke University by meeting either; 2 major, 1 major and 3 minor, or 5 minor criteria. The presence
of a new murmur fulfils a major criterion.
Haematuria and proteinuria are extremely common findings in endocarditis and are usually caused
by an immune glomerulonephritis mediated by the circulating immunoglobulins stimulated by the
infection. This may be present even when the urea and creatinine are normal. Urinalysis should be
part of the routine assessment of patients with endocarditis and is a simple non-invasive test for
monitoring progress. Persistent or recurrent haematuria may indicate failure of medical treatment.
The C reactive protein (CRP) and erythrocyte sedimentation rate (ESR) are not part of the Duke
criteria for the diagnosis of endocarditis. However, they are nearly always raised in endocarditis and
they are useful for monitoring response to treatment
A normal transoesophageal echo has a negative predictive value of over 90% in excluding
endocarditis in clinically suspected cases with a high index of suspicion. However, a single
www.AceMedicine.com

transoesophageal echo, cannot exclude endocarditis. Repeating the transoesophageal echo after a
few days provides further reassurance and is probably the most effective way of excluding
endocarditis (negative predictive value of over 95%).

Cardiology SBA 12
A 48-year old patient gives a 5 day history of dyspnoea. He is found to be in atrial fibrillation. Which
is the most appropriate management plan?
a) Chemical cardioversion with IV amiodarone
b) Chemical cardioversion with IV flecainide
c) Anticoagulation with warfarin and rate control
d) Transthoracic echocardiogram to exclude thrombus followed by DC cardioversion
e) Anticoagulation with warfarin then initiation of oral amiodarone (loading)

c). Anticoagulation with warfarin and rate control

Neither chemical or DC cardioversion should be attempted in a patient with AF that is presumed to


be >48 hours in duration without prior adequate anticoagulation. Adequate anticoagulation is
considered to have been achieved when a target INR of 2-3 has been reached for a minimum of 4
weeks. Cardioversion can be safely performed if a left atrial thrombus has been excluded on
transoesophageal echocardiography. However, transthoracic echocardiography is unable to visualize
the left atrial appendage satisfactorily and cannot exclude left atrial thrombus. Both amiodarone and
flecainide are anti-arrhythmic agents and should not be commenced until the patient is formally
anticoagulated in this setting. Even chemical cardioversion can disrupt a thrombus sitting in the left
atrial appendage leading to stroke .

Cardiology SBA 13
A patient becomes hypotensive following thrombolysis for an inferior myocardial infarction. Right
ventricular infarction is suspected. What is the most appropriate initial treatment?
a) IV nitrates
b) IV dobutamine
c) IV noradrenaline
d) IV fluids
e) Insertion of a temporary pacemaker wire

d). IV fluids

The aim of treatment in patients with haemodynamically significant right ventricular infarction is to
increase the right-sided stroke volume. If there is no evidence of associated left ventricular
dysfunction, patients should be given several litres of IV normal saline rapidly. Drugs that reduce the
pre-load, such as nitrates, opiates and diuretics, should be avoided. If volume loading does not
improve the blood pressure, then inotropes can be considered. In patients with left ventricular
dysfunction, reducing the afterload will help forward flow from the right ventricle, therefore,
www.AceMedicine.com

vasoconstrictors such as noradrenaline should be avoided where possible. Dobutamine can augment
forward flow and the cardiac output through its inotropic and chronotropic effects upon the left
ventricle. An infarcted, poorly functioning right ventricle can have its stroke volume significantly
compromised by concurrent bradyarrhythmias in which case temporary pacing is indicated.

Cardiology SBA 14
In a patient with left ventricular heart failure and classified in New York Heart Association (NYHA)
functional class II, which of the following medications is of prognostic benefit?
a) Carvedilol
b) Digoxin
c) Frusemide
d) Spironolactone
e) Isosorbide mononitrate

a). Carvedilol
Cardio-selective beta blockers (bisoprolol, carvedilol, metoprolol) have been shown to improve
morbidity (reduce admissions with LVF) and mortality in NYHA class I-IV patients. NYHA class I refers
to patients without any symptoms of dyspnoea at rest or even on exertion. NYHA classes II, III and IV
refer to patients with dyspnoea on moderate exertion, minimal activity and at rest respectively. Beta
blockers reduce the activation of the renon-angiotensin-aldosterone axis which has a deleterious
effect upon left ventricular (LV) remodeling by causing dilatation. Digoxin, nitrates and frusemide are
helpful in alleviating symptoms of LV failure, but have not been shown to offer a prognostic benefit.
Spironolactone, an aldosterone antagonist, has been shown to reduce mortality in patients with
class III-IV failure only.

Cardiology SBA 15
Which of the following investigations is the most important in evaluating a patient with confirmed
left ventricular failure and recurrent syncope?

a) Echocardiogram
b) Exercise stress test
c) Stress echocardiogram
d) Holter monitoring
e) Serum brain natriuretic peptide (BNP)

d). Holter monitoring

Patients with impaired left ventricular (LV) ejection fraction are at a much greater risk of developing
malignant ventricular arrhythmias and should be suspected in any heart failure patient who presents
with syncope. If this is confirmed on monitoring then the patient would be eligible for an
implantable cardiovertor defibrillator (ICD). None of the other tests would provide useful
www.AceMedicine.com

information in the syncopal patient with LV failure, unless their symptoms were manifesting on
exertion, in which case an exercise stress test could be considered. Patients with heart failure are
also liable to postural hypotension due to excessive diuretic therapy.
www.AceMedicine.com

CARDIOLOGY MCQ
1. Features of severe mitral regurgitation include:
A. Cardiomegaly
B. S4
C. Loud S1
D. Diastolic rumble
E. Systolic thrill over apex

A,E
Mitral regurgitation may lead to cardiomegaly as there is progressive volume overload. There may
be an S3 due to rapid ventricular filling but not usually an S4 (caused by contraction of stiff
ventricle). A loud S1 is more typically associated with mitral stenosis. There may be a systolic thrill.

2. Features of severe aortic stenosis include:


A. Collapsing pulse
B. S4
C. LBBB
D. Plateau pulse
E. Displaced apex beat

B,C,D

Severe aortic stenosis is associated with a plateau, slow rising pulse. The stiff ventricle contracting
may cause S4 to be heard. LBBB can arise due to conduction disease. The apex is usually heaving and
undisplaced.

3. Signs of cardiac tamponade include:


A. Tachycardia
B. Hypotension
C. Raised JVP that falls with deep breath
D. Pulsus paradoxus
E. Decreased urinary output

A,B,D,E

Tamponade is associated with a raised JVP that (paradoxically) does not fall with inspiration
(Kausmaul’s sign). The rest are associated with clinical tamponade.
Chris Lockhart

4. Possible causes of atrial fibrillation include:


A. Mitral stenosis
B. Thyrotoxicosis
C. Pneumonia
www.AceMedicine.com

D. Pregnancy
E. Dilated cardiomyopathy

A,B,C,D,E

These are all causes of AF, which if treated promptly and appropriately, can prevent recurrence of
the arrythmia.

5. Associations of VSD include:


A. Harsh systolic murmur
B. RBBB
C. Trisomy 21
D. Aortic coarctation
E. Pulmonary stenosis

A,B,C,D,E

All of the above are associated with a VSD

6. Features of aortic coarctation include:


A. diastolic murmur
B. diastolic tail on echocardiogram of the descending aorta
C. radio-femoral delay
D. rib-notching
E. kyphoscoliosis

B,C,D

It is typically a systolic murmur that is heard best at the back in the inter-scapular space. Rib-
notching is because of the formation of collateral vessels. Kyphoscoliosis is more likely associated
with Marfan’s syndrome.

7. Associations with constrictive pericarditis include:


A. tachycardia
B. raised JVP
C. previous radiotherapy to chest wall
D. 5-fluorouracil
E. tesicular tumors

A,B,C
www.AceMedicine.com

Pericarditis is associated with tachycardia and raised JVP due to increased atrial pressures. It may be
a complication of previous radiotherapy. 5-fluorouracil is associated with coronary vasospasm.
Testicular tumors are associated with supraventricular tachycarrythmias

8. Possible causes of Ventricular tachycardia include:


A. Ischaemic cardiomyopathy
B. Ebsteins anomaly of the tricuspid valve
C. Arrythmogenic right ventricle
D. Low potassium
E. erythromycin

A,B,C,D,E

Ebstein’s anomaly of the tricuspid valve (TV) is a condition where the TV annulus is much lower into
the ventricle (ie ventricular atrialisation) and patients are at risk of sudden death. Profound
electrolyte disturbance can cause VT. Any drug that prolongs the QT interval can cause VT.

Endocrinology SBA 1
Hypoparathyroidism
Which one of the following is associated with hypoparathyroidism?
A. Coeliac disease
B. Cystic fibrosis
C. DiGeorge syndrome
D. CREST syndrome
E. Crigler Najjar syndrome
C: DiGeorge syndrome is a familial condition, characterised by hypoparathyroidism associated with
intellectual impairment, cataracts and calcified basal ganglia.

Endocrinology SBA 2
Addison’s disease
Which of the following combination of biochemical disturbances is the classical finding in a patient
with untreated Addison’s disease?

A. Hypernatraemia, hypokalaemia, hypoglycaemia


B. Hyponatraemia, hyperkalaemia, hypoglycaemia
C. Hyponatraemia, hypokalaemia, hypoglycaemia
D. Hypernatraemia, hypokalaemia, hyperglycaemia
E. Hypernatraema, hyperkalaemia, hyperglycaemia
www.AceMedicine.com

B. Hypoadrenalism classically causes hyponatraemia, hyperkalaemia and hypoglycaemia.


Mineralocorticoid deficiency has its predominant effect on the extracellular balance of sodium and
potassium in the distal tubule of the kidney. Glucocorticoid deficiency has a major effect on
carbohydrate metabolism, with associated hypoglycaemia.

Endocrinology SBA 3
Phaeochromocytoma
Which of the following is the most appropriate initial investigation if concerned re
phaeochromocytoma?

A. 24 hour urinary 5-HIAA


B. 24 hour urinary catecholamines
C. Dexamethasone suppression test
D. Insulin tolerance test
E. Short synacthen test

B. 24hr urinary catecholamines.


The measurement of urinary metabolites is a useful screening test if concerned regarding
phaeochromocytoma. Normal levels on three 24hr collections practically exclude the diagnosis.
High concentrations of 24hr urinary 5-HIAA (major metabolite of 5-HT) are found in patients with
carcinoid tumours. The dexamethasone suppression test is useful in diagnosing Cushing’s syndrome.
The short synacthen test is helpful in diagnosis of hypoadrenalism. The insulin tolerance test is used
in the diagnosis or exclusion of ACTH and growth hormone deficiency.

Endocrinology SBA 4
Hirsutism

A 23yr old female presents with a 2month history of hirsutism, acne and deepening of her voice.
She has a markedly elevated testosterone.
What is the likely aetiology?
A. Polycystic ovarian syndrome
B. Adrenal tumour
C. Prolactinoma
D. Carcinoid syndrome
E. Wilson’s syndrome

B. Adrenal tumour.
The short duration of her symptoms, and the substantially raised testosterone would make one
suspicious of a virilising tumour. Serum testosterone may be elevated in PCOS, but not markedly so,
and the symptoms would usually be of longer duration.

Endocrinology SBA 5
Hyperaldosteronism
www.AceMedicine.com

A patient present with elevated blood pressure and low potassium. You are concerned about a
possible secondary cause for her hypertension. She is no on any medication.
Which screening investigation would be most appropriate?
A. Synacthen test
B. Chest X Ray
C. Plasma aldosterone:renin ratio
D. 24 hour urinary catecholamines
E. Clonidine suppression test
C. Plasma aldosterone:renin ratio.
The main concern in a patient with hypertension and hypokalaemia is primary hyperaldosteronism.
An elevated plasma aldosterone:renin ratio is a valuable screening test.

Endocrinology SBA 6
Hypogonadism
Which of the following is not associated with hypogonadism?
A. Loss of libido
B. Testicular atrophy
C. Osteoporosis
D. Subfertility
E. Gynaecomastia

E . Gynaecomastia.
This may be a side-effect of the treatment for hypogonadism.

Endocrinology SBA 7
Hypopituitarism
Which of the following is not a cause of hypopituitarism?

A. Pituitary adenoma
B. Kallmann’s syndrome
C. Pituitary apoplexy
D. Sarcoidosis
E. Multiple sclerosis

E. Multiple sclerosis.
There are numerous conditions causing hypopituitarism, including neoplastic causes (eg: pituitary
adenoma), congenital (eg: isolated deficiency of pituitary hormones as in Kallmann’s syndrome),
vascular (eg: pituitary apoplexy) and infiltrative causes (eg: sarcoidosis)

Endocrinology SBA 8
Pituitary tumours
www.AceMedicine.com

A patient presents with galactorrhoea and amenorrhoea. Pregnancy test is negative. MRI pituitary
confirms a pituitary mass.
Which hormone is most likely being produced in excess?

A. Prolactin
B. Testosterone
C. Dopamine
D. FSH
E. LH

E. Prolactin. This lady’s symptoms suggest hyperprolactinaemia. There are many causes of
hyperprolactinaemia, including prolactinoma, co-secretion of prolactin in acromegaly, stalk
compression due to pituitary masses, hypothyroidism, polycystic ovary syndrome and medication.
Dopamine has an antagonistic effect on prolactin production.

Endocrinology SBA 9
Complications of diabetes
Hypoglycaemia is defined when blood sugar falls below what level?
A. 2 mmol/l
B. 2.5 mmol/l
C. 3 mmol/l
D. 4 mmol/l
E. 4.5 mmol/l

D: 4mmol/l

ENDOCRINOLOGY MCQ 1
Hypoparathyroidism
A patient develops hypoparathyroidism following neck surgery. Which of the following features may
develop?
A. Convulsions
B. Circumoral numbness
C. Short PR interval on ECG
D. Diarrhoea
E. Prolonged QT interval on ECG

A, B, E- Hypoparathyroidism presents as neuromuscular irritability and neuropsychiatric


manifestations. The hypocalcaemia that results from hypoparathyroidism may be severe enough to
cause a prolonged QT interval on ECG
www.AceMedicine.com

ENDOCRINOLOGY MCQ 2
Addison’s disease
Routine investigations in patients with Addison’s disease may show:
A. hypercalcaemia
B. hyponatraemia
C. hyperkalaemia
D. hypoglycaemia
E. metabolic alkalosis

A, B, C, D- Patients typically get a metabolic acidosis, not alkalosis, in hypoadrenalism. Sodium


reabsorption in the distal tubule is linked with acid secretion. Low levels of aldosterone stimulation
of the distal tubule therefore leads to sodium wasting in urine and H+ retention in serum.

ENDOCRINOLOGY MCQ 3
Hyperaldosteronism
A patient has recently been diagnosed with Conn’s syndrome. Which of the following features may
a patient with this have?
A. hypertension
B. hyperkalaemia
C. alkalosis
D. hyponatraemia
E. hypocalcaemia

A, C- Conn’s syndrome is elevated aldosterone in the absence of activation of renin-angiotensin


system. Potential causes are adrenal adenoma, bilateral adrenal hyperplasia and adrenal carcinoma
(rare). Patients typically have a hypokalaemic alkalosis. Sodium is usually mildly elevated or normal.

ENDOCRINOLOGY MCQ 4
Phaeochromocytoma
A patient attends with panic attacks, sweating, palpitations, facial flushing and hypertension. You
suspect phaeochromocytoma.
Which of the following investigations may assist you in the diagnosis of this condition?
A. 24 hour urinary catecholamines
B. CT Adrenals
C. Clonidine suppression test
D. Dexamethasone suppression test
E. Abdominal X Ray

A, B, C- 24hr urinary catecholamines would be elevated. A CT Abdomen often helps to localise the
tumours, MRI usually shows the lesion clearly. In a clonidine suppression test, plasma
www.AceMedicine.com

catecholamines are measured before and after an oral test dose is given to the patient. A positive
test occurs if there is no reduction in plasma levels.

ENDOCRINOLOGY MCQ 5
Hirsutism
A 22yr old girl presents with hirsutism and oligomenorrhoea. She has a history of acne. Her BMI is
30. You think she likely has polycystic ovarian syndrome (PCOS).
Which of the following may be used in the management of PCOS?
A. weight loss
B. topical eflornithine
C. sulphonylurea
D. loop diuretic
E. dianette

A, B, E- Topical eflornithine is used to treat facial hirsutism. The anti-androgen, cyproterone acetate
competitively inhibits androgen production at peripheral receptors and may also reduce androgen
synthesis. This is found in combined preparation in the oral contraceptive Dianette.

ENDOCRINOLOGY MCQ 6
Hypogonadism
Which investigations may be undertaken to assist in the diagnosis of hypogonadism?
A. FSH, LH
B. Testosterone
C. PSA
D. ECG
E. Chromosomal analysis
A, B, E- Testosterone level is essential to diagnose hypogonadism. FSH and LH are important to
determine whether it is a primary pituitary cause (if FSH and LH are low). Chromosomal analysis is
important, in particular to exclude Klinefelter’s syndrome. PSA is measured when patients are
treated with testosterone.

ENDOCRINOLOGY MCQ 7
Pituitary tumours
Which of the following treatments may be used in management of a microprolactinoma?
A. bromocriptine
B. cabergoline
C. carbamazepine
D. cimetidine
E. thyroxine

A, B- A and B are dopamine agonists, therefore reducing the production of prolactin.


www.AceMedicine.com

ENDOCRINOLOGY MCQ 8
Hypopituitarism

Which of the following are possible clinical features in a patient with hypopituitarism?

A. loss of axillary hair


B. loss of libido
C. dysmenorrhoea
D. hypertension
E. bitemporal hemianopia

A, B, E- The clinical features associated with hypopituitarism are many – due to both an
underproduction of pituitary hormones and possible mass effects from a pituitary tumour. Females
may have oligo- or amen-orrhoea rather than dysmenorrhoea. It is also more likely for patients to
have hypotension than hypertension.

Rheumatology SBA on joint pain


A 25 year old lady presents with increasing joint pains. Clinical examination reveals swollen tender
DIP joints, pitting in her nails and swollen toes. Her spinal movements are also restricted. She also
describes episodes of bloody diarrhoea and weight loss over the past year which have never been
investigated.

Which of the following is the most likely cause of her joint pains?

A. Rheumatoid arthritis
B. Psoriatic arthritis
C. Fibromyalgia
D. SLE
E. Reactive arthritis

b. Psoriatic arthritis

Psoriatic arthritis (PSA) is the most likely cause as evidenced by the DIP joint involvement and the
dactylitis (swollen toes). Although she may not have the actual skin rash she has evidence of nail
psoriasis in the form of pitting. Oncholysis is another manifestation of nail psoriasis. She may well
have an associated spondylarthropathy which needs to be investigated as do her bowel symptoms -
she may have inflammatory bowel disease which can be associated with PSA and
spondylarthropathy
www.AceMedicine.com

Rheumatology MCQ on joint pain

A 45 year old lady complains of increasing widespread joint pains which are worse in the evening
after a stressful day at work. She describes puffy hands and feet and a painful neck. Her
concentration is very poor and she has recently suffered from marital problems. Rheumatoid factor
is mildly positive . Examination reveals an increased BMI and global restriction of movement due to
pain, but no synovitis. Which of the following investigations would be useful in this case?
A. Anti-CCP antibody
B. USS hands and feet
C. Anti-JO-I antibody
D. ESR and CRP
E. Anti-mitochondrial antibody

A, B and D.

This lady most likely has fibromyalgia as evidenced by the lack of objective signs and the association
with stress and family problems. However, it is always very important to exclude organic disease
before a firm diagnosis can be reached. Rheumatoid factor can be mildly positive in many of the
normal population therefore anti-CCP antibody which is more sensitive for inflammatory disease and
inflammatory markers, such as ESR and CRP would be helpful. Plain x-rays would likely be normal
even in early inflammatory disease, however USS is more sensitive at picking up early joint damage
which could be indicative of underlying inflammation.

Rheumatology SBA on back pain


A 25 year old sportsman attends with increasing low back pain over the past 3 months particularly in
the mornings. He describes no trauma or sudden initiating event. History taking reveals several
episodes of an acutely painful eye for which he needed a course of steroid eye drops on each
occasion. His sister and mother suffer from psoriasis. Neurological examination is normal, however
the pain is fairly localised to his buttocks with no radiation. He also describes several attacks of a
painful left Achilles’ tendon. What is the most likely cause of his back pain?
A. Osteoporosis
B. Prolapsed intervertebral disc
C. Sacro-iliitis
D. Infective disciitis
E. Sciatica

C. Sacro-iliitis
This patient describes several features which are in keeping with the sero-negative group of
arthridites - Sacro-ilitiis, psoriasis and iritis. Even a family history of these conditions is an
association to be noted. Localised buttock pain is fairly typical of sacro-ilietis. The pain of sciatica
would be expected to radiate while both infective discitis and a prolapsed disc should also produce
some pain in the spine. There might also be systemic upset with infection. Osteoporosis itself
should not give pain unless there is an associated collapsed vertebra.
www.AceMedicine.com

Rheumatology MCQ on back pain


A 50 year old lady develops sudden mid-thoracic pain while coughing. The GP suspects a fractured
rib so she attends for an x-ray. Her ribs look fine, however there is an abnormality at the level of her
seventh thoracic vertebrae. She has a history of anorexia nervosa and although she has now
recovered her BMI remains 18. She is not on any regular medication. Which of the following would
be appropriate treatments in this case?
A. Steroids
B. Physiotherapy
C. Bisphosphonates
D. Opiate analgesia
E. High fat diet

C and d .
This lady most likely has suffered a collapsed vertebrae as a result of underlying osteoporosis. Risk
factors for osteoporosis include steroid use, coeliac disease, anorexia nervosa, low body weight,
early menopause, positive family history, smoking and alcohol. A young age of this patient suggests
that she is suffering from secondary osteoporosis. The acute pain of a collapsed vertebrae is very
severe so she will need strong analgesia and in the longer term will need a DEXA scan and
bisphosphonate treatment.

Rheumatology SBA on RA
A 30 year old girl with known seropositive RA is maintained on methotrexate 20mg weekly and is
doing well. She is keen to start a family.
What is the best advice to give her?
A. Continue with methotrexate and add oral steroids
B. Stop methotrexate and start leflunamide
C. Stop methotrexate and start cyclosporin
D. Stop methotrexate and give oral steroids if necessary for disease flares
E. Reduce the dose of methotrexate

D- Stop methotrexate and give oral steroids if necessary for disease flares.
Methotrexate is a known teratogenic agent and therefore is not safe in pregnancy or prior to
conception. The current advice is that it should be stopped for at least several months before
conception. Obviously this may results in disease flares so therefore steroids can be used if
necessary. Although there is limited data on a lot of the DMARDS, only sulfasalazine is thought to be
probably safe if used with folate supplementation. Low dose steroids are relatively safe although
high doses are associated with a small risk of cleft palate.
www.AceMedicine.com

Rheumatology MCQ on RA
A 70 year old woman has an 8 week history of increasing small joint pain and swelling with
associated weight loss and fatigue. She is very stiff especially in the mornings and complains of
severe neck pain. She has painful soft tissue lumps over the extensor aspect of her upper forearms
and tingling affecting her fingers which wakes her at night. Her rheumatoid factor is strongly
positive. X-rays show erosions at her MCP joints. Which of the following are diagnostic criteria for
rheumatoid arthritis?
A. Subcutaneous nodules
B. Positive rheumatoid factor
C. Weight loss
D. Erosions on x-ray
E. Carpal tunnel syndrome

A, b and d.
The 1987 ACR revised criteria for the classification of RA include morning stiffness > 1 hour, swelling
of three or more joint areas, swelling of the PIP, MCP or wrist joints, symmetric arthritis, positive
rheumatoid factor, subcutaneous nodules and erosions or peri-articular osteopenia or x-ray.
Patients must have four out of six criteria. Neck pain, weight loss, fatigue, carpal tunnel syndrome
are all associated features, but are not diagnostic criteria.

Rheumatology SBA on septic arthritis


A 40 year old man with known rheumatoid arthritis presents with an acutely hot swollen knee which
is extremely painful. ESR is 70mm/hr and CRP is > 200 mg/l. He is vomiting and pyrexic with a temp
of 39 degrees. Joint aspiration yields 60 mls of turbid thick fluid. What is the most appropriate anti-
microbial treatment?
A. IV flucloxacillin and oral fusidic acid
B. IV co-amoxiclolav
C. IV clindamycin
D. Oral flucloxacillin and oral fusidic acid
E. IV piperacillin

A- IV flucloxacillin and oral fusidic acid.


Most septic joints are caused by either staphylococcus or streptococcus organisms therefore these
would be the most appropriate antibiotics. Fuscidic acid is absorbed as effectively in oral form as
long as the patient is not vomiting and can tolerate it. Flucloxacillin should be given intravenously in
the first instance. Clindamycin is useful for patients who are penicillin allergic. Piperacillin is second
line and may be used on the advice of the microbiologist if the patient is not responding or if
antibiotic sensitivities are known.
www.AceMedicine.com

Rheumatology MCQ on septic arthritis


A 25 year old male presents to Casualty with an acutely painful hot swollen ankle. There is no
history of arthritis. He did however sustain a deep laceration to his calf a few days previously. He is
pyrexic with rigors, his pulse is 120bpm and his blood pressure is 80/40mmHg. IV fluids have been
started and paracetamol has been given. You are asked what immediate investigations are needed.
A. Immediate joint aspiration for culture and microscopy
B. ESR and CRP
C. Blood cultures
D. Rheumatoid factor
E. Throat swab

A, b and c.
This patient is clearly very unwell and is severely haemodynamically compromised. The laceration is
very likely the portal of entry for the infection which is causing severe systemic upset. It is very
important to aspirate the joint in order to culture any organisms so that the correct antibiotics can
be administered once sensitivities are known. Broad spectrum antibiotics can be started until
sensitivities are known. Blood cultures will also be useful as the organisms may also be present in
the blood (septicaemia). Inflammatory markers (ESR, CRP) will be useful to monitor the situation.
Rheumatoid factor will not be particularly, helpful neither is there any indication for a throat swab.

Rheumatology MCQ on osteoarthritis

A 50 year old lady presents with increasing pain in her left knee particularly when descending stairs.
There is no obvious swelling, but there is notable crepitus on examination. She also has bony
swellings at her PIP and DIP joints. Her BMI is 32 and she is a heavy smoker who drinks occasional
alcohol. Her mother has bilateral hip replacements. Which of the following are risk factors for
osteoarthritis (OA)?

A. Positive family history


B. Old surgery/trauma to the joint
C. Obesity
D. Alcohol excess
E. Osteoporosis

A, b and c.
Although the precise genetics of OA are not yet entirely known, initial work has shown a definite
hereditary element to the disease so family history is definitely relevant and it is not solely a ‘wear
and tear’ disease. Previous trauma or surgery to a joint makes it more pre-disposed to developing
osteoarthritis as the original joint architecture has been disrupted. Obesity places extra
stress/loading on the joints and therefore makes cartilage loss and subsequent OA more common.
There is no definite evidence that alcohol increases the risk. Osteoporosis is a different disease
which causes a loss of bone density.
www.AceMedicine.com

Rheumatology SBA on osteoarthritis


A 60 year old teacher presents with widespread joint pain. Her hands are particularly painful
notably the small joints in her fingers and the joint at the base of her thumb. Her wrist is also painful
although she has a scar from surgery to repair a Colles’ fracture 20 years ago. She also describes left
groin and knee pain when walking and also low back pain. She has difficultly finding shoes to fit due
to ‘bunions’ bilaterally. Which of the following joints are NOT affected by primary osteoarthritis?
A. DIP joints
B. CMC joints
C. Wrist
D. Hip
E. MTP

C- Wrist.
Primary or idiopathic OA affects the DIP and CMC joints in the hands, MTP joints in the feet, spine,
acromio-clavicular shoulder joint and also the knee and hip. If any other joints are affected by OA
then a secondary cause should be searched for. Secondary causes include congenital disorders or
dysplasias, joint hypermobility syndromes, previous trauma, metabolic diseases such as
haemachromatosis or crystal deposition diseases, endocrine disorders such as acromegaly or
hyperparathyroidism, neuropathic joints, previous infections or inflammatory disorders.

Rheumatology SBA on crystal arthropathies


A 50 year old man presents with an acutely swollen knee. His CRP is 200mg/ l . He is apyrexic.
Aspiration yields 2mls of turbid fluid. Negatively birefringent needle shaped crystals are seen on
microscopy and there are no organisms seen. He is in severe pain, however his renal function shows
a creatinine of 300µmol/l. His INR is normal. What is the best course of action?
A. NSAIDS
B. Start allopurinol and colchicine
C. Opiate analgesia
D. Intra-articular steroid injection
E. Oral steroids

D. Intra-articular steroid injection.

The appearance of the negatively birefringent needle shaped crystals give a diagnosis of gout which
is in keeping with the clinical features. His poor renal function limits the treatment options. NSAID’s
are contraindicated as they could precipitate renal failure. Allopurinol should not be started straight
away anyway, however it will need to be used with caution (if at all) with this degree of renal
impairment. Likewise colchicine can cause diarrhoea which could lead to dehydration which again
would not be a sensible risk to take with this patient. Steroids are useful to settle the acute
inflammation and giving them locally directly into the joint would be more appropriate as it would
reduce their systemic absorption and hopefully act more quickly. Opiate analgesia could be used
sparingly, however it will not settle the inflammation cause by the crystal deposition.
www.AceMedicine.com

Rheumatology SBA on autoimmune CT diseases


A 40 year old solicitor presents feeling ‘out of sorts’. She describes cold hands and feet and the
typical colour changes of Raynaud’s phenomonen. On examination however, tightening and
thickening of her skin is noted from her fingertips extending up to below her shoulders. Her face is
not affected. She also describes increasing shortness of breath on minimal exertion. Urinalysis
shows proteinuria and haematuria and she is hypertensive. A recent auto-antibody screen shows a
positive anti-topoisomerase (anti-Scl-70) antibody.Which of the following is the most likely
diagnosis?
A. Limited cutaneous scleroderma
B. Diffuse systemic sclerosis
C. Limited systemic sclerosis
D. SLE
E. Sjogrens disease

B. Diffuse systemic sclerosis.


Systemic sclerosis (SSc) is a connective tissue disease characterised by progressive vascular
dysfunction, auto-immune inflammation and finally fibrosis which is often irreversible. The skin and
internal organs are affected. It is classified into several groups. Limited systemic sclerosis involves
the face and the skin distal to the elbows and knees only, while diffuse systemic sclerosis involves
skin both proximal and distal to the elbows and knees. Unfortunately it carries a worse prognosis
and more often is associated with kidney disease (manifesting in this patient as proteinuria and
haematuria). Urgent attention needs to be given to the kidney disease as patients can develop an
acute renal crisis accompanied by severe hypertension. Mortality is high in these cases.
Lung fibrosis can present in both groups although it is more common in diffuse SSc. Anti- SCL -70
antibody is more commonly associated with diffuse SSc, whilst anti-centromere antibody is more
common in limited SSc.

Rheumatology MCQ on autoimmune CT diseases


A 35 year old plumber presents with difficulty swallowing and shortness of breath. He is unable to
rise from a squatting position and so finds difficulty with his job. He has discolouration around his
eyes and on his hands. Examination reveals grade 3 muscle weakness in his thighs.
Which of the following would be helpful in diagnosis in this case?
A. Anti- Jo-1 antibody
B. HRCT chest and PFT’s
C. Anti-Ro antibody
D. Muscle biopsy
E. Anti-LA antibody

A, B and D.
The most likely diagnosis in this case is dermatomyositis. This is an autoimmune inflammatory
disease of muscle which typically causes proximal muscle weakness. There are associated skin
manifestations in the form of purplish eyelid discolouration and papules over the knuckles
(Gottron’s papules). It can be associated with lung fibrosis so imaging of the lungs is needed in this
www.AceMedicine.com

case.. A muscle biopsy would be important in order to get a definite histological diagnosis which is
useful for prognostic purposes. Many cases are associated with the anti-Jo-I antibodies which are
anti-synthetase antibodies. CK levels are also usually raised, which reflects the muscle inflammation.

Rheumatology MCQ on DMARD side-effects


A 35 year old lady has a history of poorly controlled rheumatoid arthritis. She has previously been
treated with salazopyrin, leflunamide and gold. She is now doing well on a combination of
methotrexate and hydroxychloroquine. As a result her prednisolone dose is being gradually
reduced. She has had a persistently low white cell count for several years, which needs constant
monitoring. Her most recent results show a white cell count of 2.5 x 109/L and a neutrophil count of
0.5 x 109/L . Which of the following could have contributed to her neutropenia over the years?
A. Methotrexate
B. Leflunamide
C. Prednisolone
D. Diclofenac
E. Salazopyrin

A,B ,E.
Unfortunately neutropenia is a side-effect of many of the DMARD’s. Patients must be informed of
this and must undergo regular blood monitoring. If the white cell count is persistently low, but
stable, and if the patient is not suffering from regular infections then they may usually remain on the
treatment. If however the cell count continues to drop or they get repeated infections then the drug
must be stopped. Prednisolone is a steroid which does not usually affect the white cell count.
Diclofenac is a NSAID.

Rheumatology SBA on DMARD side-effects


A 60 year old lady with known rheumatoid arthritis attends the clinic with her daughter. She is doing
very well with no inflammation and leads an active life. She has been maintained on the same
therapy for over 20 years. Her daughter reports that she has developed a blue-grey discolouration
on her face gradually over the past year and it has become more noticeable. Otherwise she is well
with no other side-effects. She attends her GP practice for regular blood monitoring which has been
satisfactory. Which of the following DMARDS could be responsible for her skin discolouration?
A. IM gold
B. Methotrexate
C. Leflunamide
D. Azathioprine
E. Salazopyrin

A) IM gold
www.AceMedicine.com

This skin discolouration is known as Chrysiasis and develops as a result of prolonged IM gold therapy.
It is usually asymptomatic apart from the obvious cosmetic appearances. Often if patients are
achieving good disease control with the IM gold they are willing to accept the skin discolouration.

Rheumatology MCQ on vasculitis


A 30 year old male attends A and E with fever, abdominal pain and arthralgia. He is severely
hypertensive with haematuria and proteinuria on urinalysis. He has a purpuric rash over his legs
and trunk and also admits to severe testicular pain. ESR >100mm/hr, CRP is normal. An urgent
vasculitic screen shows a negative p ANCA and c ANCA. A subsequent renal angiogram shows renal
artery aneurysms. What is the most likely diagnosis?
A. Wegener’s granulomatosis
B. Polyarteritis nodosa (PAN)
C. Takayasu’s artertitis
D. Polymyalgia rheumatica
E. Microscopic polyangiitis

b. PAN
Pan is a vasculitis affecting medium sized vessels. It typically causes constitutional symptoms. The
most serious effects are as a result of aneurysm formation and can affect blood vessels in the brain,
kidney or heart causing organ failure and subsequent death if not treated. ESR will be non-
specifically raised in any vasculitic disease. CRP is usually normal in vasculitis unless there is co-
existing infection. cANCA is usually indicative of Wegener’s granulomatosis and p ANCA (although
less specific) can be positive in microscopic polyangiitis.

Rheumatology SBA on vasculitis


A 30 year old secretary presents with haemoptysis. She also has a vasculitic rash on her legs where
some areas are coalescing and forming ulcers. She has a background of sinusitis for the past few
years. Her ESR is 90mm/hr. She has proteinuria on dipstick urinalysis. A chest-x-ray shows multiple
focal opacities in the lungs. An autoimmune screen shows a negative anti-nuclear antibody,
negative anti-ds DNA, negative rheumatoid factor but a positive cANCA screen. What is the most
likely diagnosis?
A. Polymyalgia rheumatica
B. Wegener’s granulomatosis
C. Microscopic polyangiitis
D. Kawasaki’s disease
E. Churg-Strauss vasculitis

b. Wegener’s granulomatosis
Wegener’s granulomatosis is a small vessel vasculitis which can affect any organ and is fatal if not
treated. Typically the respiratory tract can be affected causing sinusitis, rhinitis, epistaxis, otitis
media or haemoptysis. The formation of necrotising granulomata is typical of the disease (likely in
the lungs in this case). Glomerulonephritis is another serious manifestation which needs urgent
treatment. A vasculitic screen will typically show a positive c ANCA. Treatment with high dose
www.AceMedicine.com

steroids initially is needed followed by strong immunosuppression in the longer terms usually in the
form of cyclophosphamide or mycophenalate. Polymyalgia is an inflammatory syndrome which
usually affects older patients and causes pain and stiffness in the shoulder and/or pelvic girdles. It
usually responds rapidly to steroid therapy. Kawasaki’s disease is a systemic vasculitis presenting
usually in young children.

Rheumatology MCQ on spondylarthropies


A 45 year old solicitor attends for his routine out-patient clinic review. He has a diagnosis of
ankylosing spondylitis with peripheral joint involvement and is maintained on salazopyrin and
regular anti-inflammatory drugs. He is doing well from the point of view of his joints, however
reports increasing shortness of breath on minimal exertion to the extent that he can no longer
exercise for any length of time. He also describes episodes of a ‘red eye’. Clinical examination of the
precordium detects a heart murmur and on auscultation of his chest fine bibasal crackles are heard.
A chest-x-ray, echocardiogram and pulmonary function tests are requeseted in the first instance.
Which of the following can be associated with ankylosing spondylitis?
A. Aortic stenosis
B. Aortic regurgitation
C. Apical lung fibrosis
D. Anterior uveitis
E. Episcleritis

B,C and D.
Ankylosing spondylitis can be associated with several extra-articular features and this must be borne
in mind when treating these patients. Aortic regurgitation (AR) is the associated valvular
abnormality. His dyspnoea could be related to his AR or he may indeed have a degree of lung
fibrosis which needs to be investigated with a chest x-ray and probably further imaging in the form
of HRCT of the chest. Anterior uveitis is associated with the group of spondylarthropathies, namely
ankylosing spondylitis, psoriatic arthritis, Reiter’s disease

Rheumatology SBA on SPOND


A 30 year old male teacher was diagnosed with ankylosing spondylitis when he was 22 year old. He
had been maintained on regular anti-inflammatory drugs and was taking regular exercise in the form
of swimming and football. Over the past year however the pain and stiffness in his spine (thoracic,
cervical and lumbar) has been increasing and he is finding difficulty driving as his neck movements
are becoming more restricted. His early morning stiffness lasts until lunchtime which makes teaching
very difficult. He has tried several alternative anti-inflammatory drugs with no benefit. He has no
other joint involvement and is otherwise healthy. What is the most appropriate course of action?
A. Commence anti-TNF treatment
B. Start methotrexate
C. Start oral steroids
D. Regular hydrotherapy
E. Add an additional NSAID
www.AceMedicine.com

A- Commence anti-TNF treatment.


This patient’s disease is clearly progressing and is affecting his quality of life. There is no evidence
for methotrexate for the treatment of spinal disease in ankylosing spondylitis – it is only useful if
there is peripheral joint involvement. Steroids may help in the short term but are not advisable in
the long term due to their many side-effects. Again hydrotherapy and physiotherapy are definitely
beneficial, but this patient is already swimming and exercising to no avail. It is not advisable to take
any more than one NSAID at any one time. Anti-TNF treatment is therefore the most appropriate
and has proven to be very beneficial in these patients.

Rheumatology EMQ 1
Causes of joint pain

a)Rheumatoid arthritis
b)Septic arthritis
c)Gout
d)Haemarthrosis
e)Chondrocalcinosis
f) Psoriatic arthritis
g)Osteoarthritis
h)Reactive arthritis
i)SLE
j)Ankylosing spondylitis
k)Fibromyalgia
l)Myositis

Which of the above is the most likely cause of joint pain in each case:

Q1. A 50 year old man presents with an acutely swollen red hot 1st MTP joint. His serum creatinine is
252 µmol/l and he drinks half a bottle of wine daily.

c) Gout. Predisposing factors are male sex, alcohol, poor kidney function, diabetes, diuretic use.
Typically extremity joints such as the toes are affected when the uric acid crystallises to form urate
crystals which provoke an inflammatory reaction.

Q2. A 70 year old woman presents with a hot swollen knee. There is no history of trauma. She is on
warfarin for atrial fibrillation and her INR is > 7. She is apyrexic and inflammatory markers are
normal.

d) Haemarthrosis. Spontaneous bleeding is common with a very elevated INR. Aspiration will yield
frank blood. Absence of a temperature and normal inflammatory markers makes sepsis less likely.
www.AceMedicine.com

Q3. A 42 year old lady is referred with fatigue, low mood, palpitations and widespread joint pains.
There is no obvious synovitis on examination, but her muscles are very tender. Blood tests are
normal.

k) Fibromyalgia. Fibromyalgia is the most likely although it is important to exclude other causes.
Normal blood tests and clinical examination make inflammatory joint disease less likely. The other
symptoms such as tender muscles (‘trigger points’), fatigue, low mood and palpitations are also
typical of fibromyalgia.

Q4. A 23 year old girl complains of increasing morning stiffness. She is unable to play the piano
because of bilateral MCP joint swelling . Her ESR is 52 mm/h and CRP 45 mg/l. Autoimmune screen is
normal. Rheumatoid factor is positive.

a) Rheumatoid arthritis (RA) is likely because of the stiffness and small joint symmetrical hand
swelling. Psoriatic arthritis is an important differential diagnosis, but it is often asymmetrical and
also can affect DIP joints. Elevated inflammatory markers are also in keeping with RA as is the
positive rheumatoid factor.

Q5. A 70 year old woman is recovering from pneumonia in hospital. She develops a painful swollen
wrist. She is apyrexic. Positively birefringent rhomboid shaped crystals are aspirated.

e) Chrondrocalcinosis. Chondrocalcinosis (pseudogout ) is very common is patients (especially the


elderly) with an intercurrent illness such as diarrhoea or pneumonia. Wrists and knees are joints
commonly affected and aspiration yields calcium pyrophosphate dehydrate (CPPD) crystals as
opposed to gout which is caused by needle-shaped negatively birefringent urate crystals.

Rheumatology EMQ 2
Which of the following are the most likely causes of joint pain in each case?

a)Juvenile idiopathic arthritis


b)Septic arthritis
c)Scleroderma
d)Osteoporosis
e)Chondrocalcinosis
f)Psoriatic arthritis
g)Osteoarthritis
h)Reiter’s syndrome
i)SLE
j)Ankylosing spondylitis
k)fibromyalgia
l)myositis
www.AceMedicine.com

Q1. A 50 year old diabetic man presents with rigors and a hot swollen knee, CRP > 200 mg/l,
Temperature: 38 degrees. He is unable to weight bear.

b) Septic arthritis. Infection must be considered because of the elevated temperature, rigors and
high CRP. Diabetes is an added risk factor for sepsis. He will need immediate joint aspiration and IV
antibiotics to prevent systemic sepsis and permanent joint damage.

Q2. A 25 year old man presents with a swollen knee. He also has ‘scabby’ feet for over 3 months
and recalls pain on urination on several occasions. He is sexually active with several partners and
also describes episodes of a ‘pink-eye’ within the last few weeks.

h) Reiter’s syndrome. Reiter’s syndrome comprises the triad of arthritis, conjunctivitis and urethritis.
Chlamydia is an important organism to be considered. Keratoderma blenorrhagica or ‘scabby feet’
can occur in up to 10% of cases.

Q3. A 22 year old man develops a swollen right ankle. Further questioning reveals he suffers from
back and neck pain which improves with exercise. Spinal x-rays show loss of the normal lumbar
lordosis and ‘bony bridges’ between the vertebrae. He also has an early diastolic murmur on
auscultation of his precordium.
j). Ankylosing spondylitis. Ankylosing spondylitis typically affects young males and peripheral joint
involvement is common. Usually the back pain and stiffness are eased by physiotherapy and an
exercise programme. Radiological features include sacro-iliitis, loss of lumbar lordosis and ‘bamboo
– spine’ appearance caused by calcification of spinal ligaments. Aortic regurgitation is one of the less
common extra-articular features.

Q4. A 25 year old female presents with two swollen toes on the left foot, a swollen ankle and wrist.
She has also suffered from episodes of iritis in the past and has been diagnosed with colitis following
a recent colonoscopy. Her sister and mother suffer from psoriasis.

f). Psoriatic arthritis. This patient describes dactylitis and has a positive family history of psoriasis.
She also has other features consistent with the sero-negative group of arthridites - iritis and colitis.

Q5. A 40 year old lady has painful swollen MCP joints on the left hand and a swollen right wrist. She
as a photosensitive facial rash. Urinalysis reveals significant proteinuria. Rheumatoid factor is
negative, but anti-nuclear antibody (ANA) and anti-ds DNA antibodies are positive.

i). SLE. This lady fulfils the criteria for the diagnosis of SLE as she has five of the eleven ACR
(American College of Rheumatology) diagnostic criteria – namely arthritis, photosensitivity, renal
involvement (proteinuria), positive anti-ds DNA and ANA. Four are needed for a diagnosis of SLE.
www.AceMedicine.com

Rheumatology EMQ 3
Causes of Hip Pain

Which of the following are the most likely causes of hip pain in each case?

a)osteoarthritis
b)septic Hip
c)trochanteric bursitis
d)synovitis
e)iliopsoas abscess
f)infective sacro-iliitis
g)Loosening of a prosthesis
h)referred pain from the knee
i)Fractured neck of femur
j)Avascuar necrosis

Q1. A 60 year old farmer is suffering from increasing hip pain over several months. X-ray shows loss
of joint space. Clinical examination reveals reduced movement overall, but total loss of internal and
external rotation. He also has bony swelling at the PIP and DIP joints in his hands.

a). Osteoarthritis (OA). The more chronic nature of the pain makes infection less likely. Typical x-
ray features of OA are loss of joint space, osteophyte formation and sub-chondral sclerosis. Internal
and external rotation are usually the first movements to be affected. He has evidence of widespread
OA as he has Heberden’s (DIP) and Bouchard’s (PIP) nodes in his hands.

Q2. A 45 year old lady complains of right sided hip pain especially when walking and lying in bed on
the affected side. Hip movements are good, but she is locally tender to palpation over her left
lateral thigh.

c). Trochanteric bursitis. Trochanteric bursitis typically affects middle aged female patients. It
occurs as a result of inflammation of the bursa overlying the greater trochanter of the femur. Pain is
typically fairly localised to the area and the hip joint itself is not affected. Local steroid injection can
be of benefit.

Q3. An 18 year old girl complains of sudden onset right hip pain. All movements are severely
reduced and any attempt to move results in severe pain. She is pyrexic with strongly elevated
inflammatory markers. X-ray is normal. Ultrasound shows a large joint effusion.

b). Septic Hip. A Septic hip must be considered given the sudden onset of the pain, the systemic
upset (fever and raised inflammatory markers) and the severity of the pain. A large effusion on
ultrasound is a clue that there is either inflammation or infection and infection seems more likely in
this case. She needs urgent aspiration and IV antibiotics.
www.AceMedicine.com

Q4. A 58 year old lady has been on long term steroids for many years because of COPD. She
develops sub-acute right hip pain. Her temperature is normal. X-ray shows total destruction of the
femoral head.

j). Avascular necrosis. The history is important here. Patients on long term steroids are at risk of
many complications, including problems with their bones. Both osteoporosis and avascular necrosis
are side-effects. In this case avascular necrosis is the cause as her femoral head has been totally
destroyed. She will be in severe pain, but should be systemically well.

Q5. A 30 year old girl with rheumatoid arthritis develops severe left hip pain. Her temperature is
normal. X-ray is normal. Ultrasound shows a large effusion and aspiration yields 20 mls clear straw-
coloured fluid which cultures no organisms.

d). Synovitis. This girl’s rheumatoid disease has involved her hip joint although infection needs to be
excluded. Again the effusion is a clue to either inflammation or infection however aspiration was
sterile (no organisms) so infection can be safely excluded. A local steroid injection may be of
benefit.

Rheumatology EMQ 4
Treatment of back pain

a)physiotherapy, muscle relaxants and analgesia


b)IV antibiotics
c)emergency radiotherapy
d)emergency spinal decompression
e)Exercise and NSAIDS
f)local steroid injectiong
g)hydrotherapy
h)Opiate analgesia followed by bisphosphonates
i)Emergency repair of AAA

Q1 A 25 year old male patient suffers from back pain which is worse in the morning and after
periods of rest. Spinal x-rays show a ‘bamboo spine’ appearance. He has also suffered from iritis in
the past and has patches of a dry scaly rash on his skin.

e). Exercise and NSAIDS. This young man most likely has ankylosing spondylitis which typically gives
pain and stiffness which is worse after rest and improves with exercise. Ankylosis or bony fusion of
the vertebrae give rise to a straight ‘bamboo-like’ appearance on x-ray. He also has other symptoms
which would fit into the sero-negative group of arthritides - iritis and psoriasis.
Physiotherapy/Exercise and NSAIDS (in the first instance) are the mainstay of treatment.
www.AceMedicine.com

Q2 A 50 year old diabetic patient develops sudden acute severe pain in her mid-thoracic region. She
is pyrexic with a CRP > 200mg/l. X-ray is normal but an MRI scan shows destruction of the T7
vertebra and evidence of infection within the disc.

b). IV antibiotics. This patient has discitis which is infection of an intervertebral disc. It can cause
severe systemic sepsis and needs urgent IV antibiotics and supportive care. Surrounding bony
destruction is common in severe infection as is abscess formation. Diabetics are particularly
susceptible with staphylococcal infection being the most likely organism. Those who have
undergone recent spinal intervention, such as spinal anaesthesia, are also at risk.

Q3. A 60 year old man with prostatic cancer develops sudden pain in his lower back with loss of
bladder control and weak legs.

d). Urgent MRI and neurosurgical consultation. This patient may well have developed cauda equine
syndrome as he is displaying several of the ‘red flags’ of back pain. His history of prostatic cancer is
important as he may have undiagnosed bony/spinal metastatic deposits and he needs urgent
decompression.

Q4. A 72 year old woman develops sudden mid-thoracic pain while lifting her grandson. A recent
DEXA scan had shown T scores of -4.5 in her spine.

h).Opiate analgesia and bisphosphonates. This lady has severe osteoporosis as evidenced by her
bone density scan results (< -2.5 ). As a results she is at high risk for a non-traumatic vertebral
fracture which is likely what has happened in this case. She will need acute pain relief followed by
bisphosphonates in the longer term to prevent further bone loss.

Q5. A 42 year old man who was previously well develops sudden back pain while bending over. He is
systemically well. There are no neurological signs on examination.

a). Initial rest, muscle relaxants and analgesia, followed by physiotherapy. A prolapsed or ‘slipped’
intervertebral disc is the most likely cause in this case.
Dr Elisabeth Ball

Rheumatology EMQ 5
Autoantibodies of connective tissue disorders

Which of the following tests could help in diagnosis in each of the following?

a)Anti-ds DNA
b)Anti-Ro
C)Anti-Ro and LA
d)Anti-RNP
www.AceMedicine.com

e)Anti-nuclear antibody
f)CRP
G)rheumatoid factor
h)anti-CCP antibody
i)Anti-JO – 1
J)ESR
K)anti-mitochondrial antibody
l)anti-SCL 70
m)anti-centromere antibody

Q1 A 32 year old girl presents with a butterfly rash on her face, painful swollen finger joints,
proteinuria and fatigue. ESR is 60mm/h, CRP is 5mg/l.

a). Anti-ds DNA is fairly specific for SLE for which this patient displays characteristic symptoms –
butterfly rash, arthritis/arthralgia, renal involvement and fatigue. ESR is typically elevated while CRP
is low unless there is co-existing infection. Anti-nuclear amtibody may also be positive although it is
not specific for SLE.

Q2. A 40 year old lady complains of sore joints. Further questioning reveals a history of dry eyes and
a dry mouth.

c).Anti - Ro and La antibodies are typically found in Sjogren’s disease. The underlying pathology
involves lymphocytic infiltration of salivary and lacrimal glands interfering with saliva and tear
production. Infiltration of other organs, such as the lungs and GI tract can result in a variety of other
less common manifestations.

Q3. A 50 year old man has been diagnosed with pulmonary fibrosis after a HRCT chest. He also has
proximal muscle weakness which responds to steroids and large ‘mechanics’ hands.

i). Anti Jo-1 antibodies are found in myositis (giving rise to proximal muscle weakness). The
syndrome of myositis, pulmonary fibrosis, anti-Jo-1 antibodies and ‘mechanics hands’ is known an
anti-synthetase syndrome.

Q4. A 47 year old lady suffers from painful colour changes of her hands precipitated by cold. She
also has difficulty swallowing and has noticed tightness of her skin especially on her face and her
fingers.

m). Anti-centromere antibody. These symptoms are typical of limited scleroderma (previously
known as CREST). Raynaud’s syndrome, oesophageal motility problems and sclerodactly are the
features she exhibits. Anti-centromere antibodies are fairly specific to limited scleroderma. Anti-
SCL-70 (anti-topoisomerase) antibodies are found in diffuse scleroderma.
www.AceMedicine.com

Q5. A 33 year old lady is under investigation for arthritis. She has a strong family history of
rheumatoid arthritis (RA) affecting her mother and sister, and her own joint involvement is typical of
RA with symmetrical small joints affected. However, her Rheumatoid factor antibody is negative as is
her anti-nuclear antibody and anti-ds DNA antibody.

h). anti-CCP antibody is a more recently discovered antibody which is much more specific for
rheumatoid arthritis than rheumatoid factor which can be falsely positive or negative. The absence
of ANA and anti-ds DNA make an auto-immune disease less likely. It is anticipated that her anti-CCP
antibody would be positive as it is likely that she is indeed suffering from RA.

Rheumatology EMQ 6
Which of the following is the most likely cause/association of Raynauds’ phenomonen?

A. Limited Scleroderma
B. Cryoglobulinaemia
C. Polycythaemia
D. Primary Raynaud’s
E. Beta blockers
F. Thiazide diuretics
G. ACE inhibitors
H. SLE
I. Polymyositis
J. Gout
K. Diffuse scleroderma

Q1. A 30 year old lady suffers from colour changes in her hands and feet secondary to the cold. She
has also notices small white deposits under the skin on the tips of her fingers. She has difficulty
swallowing. Her serology shows positive anti-centromere antibodies.

a). Limited scleroderma. Raynaud’s in a common presenting feature in up to 70% of cases. Other
features include calcinosis (hard white deposits), oesophageal dysmotility, sclerodactaly and
telangectasia. Anti-centromere antibody is positive in 50-90% of cases of limited scleroderma.

Q2. A 22 year old student notices increasingly painful colour changes in her hands and feet to the
extent that she cannot go outside without gloves or fur-lined boots. Even putting her hand in the
freezer precipitates the colour changes. She is otherwise very well with no associated symptoms.
She has no family history of arthritis or connective tissue disease and all her blood tests and
immunology tests are normal.

d). Primary Raynaud’s or ‘Raynauds disease’. Primary Raynaud’s is the most likely in the absence of
any other symptoms or positive antibodies.
www.AceMedicine.com

Q3. A 40 year old lady developed Raynaud’s fairly acutely. She has also noticed palpable purpura on
her skin. A small ulcer has developed at the tip of her 3rd finger. Her joints are generally painful and
she also has proteinuria on urinalysis.

b). Cryoglobulinaemia. Cryoglobulinaemia results from the presence of cryoglobulins which are
immunoglobulin complexes which precipitate and form a gel at low temperatures. Raynaud’s
syndrome and digital and leg ulceration are common presentations; as are glomerulonephritis,
arthritis and neuropathy. There are three different types depending on the type of cryoglobulin
isolated from the blood.

Q4. A 42 year old woman develops Raynaud’s since her anti-hypertensive medication was adjusted.
She takes a beta-blocker, a thiazide diuretic and an ACE inhibitor.

e ). Beta blockers. Beta blockers can cause Raynaud’s as a results of vasoconstriction. Other drugs
which can be precipitating agents are vinblastine, bleomycin, cyclosporine, cisplatin, interferon-
alpha.

Q5. A 50 year old man complains of abdominal pain. He describes symptoms of Raynaud’s
phenomonen. Examination reveals splenomegaly and also bruising on his skin. Laboratory
investigations show a raised haemoglobin, haematocrit, red cell count and whole blood viscosity.
c). Polycythaemia. Hyperviscosity which in turn restricts blood flow is a cause of Raynaud’s
phenomonen. Polycythaemia refers to an increase in red cell count, haematocrit and haemoglobin.
It can be primary (Polycythaemia Rubra vera) or secondary (due to increased erythropoietin levels
either from hypoxia or increased production by the kidney). This patient is more likely to have PRV
as evidenced by the splenomegaly and the Raynaud’s syndrome.

Rheumatology EMQ 7
Causes of proximal myopathy

a)polymyositis
b)Alcohol use
c)thyrotoxicosis
d)Polymyalgia rheumatica
e)McArdle’s syndrome
f)dermatomyositis
g)osteomalacia
h)Cushing’s syndrome
i)hypothyroidism
j)Amiodarone
k)myotonic dystrophy
l)osteoporosis
www.AceMedicine.com

Q1 A 45 year old man develops weakness of his legs – examination reveals proximal myopathy, a
rash over his knuckles and a purple discolouration around his eyes. He reports difficulty swallowing.
CK levels are >3,000iu/l and his anti-Jo-1 antibody is positive.

f). Dermatomyositis. This patient has all the features of this inflammatory myopathy.
Dermatomyositis differs from polymyositis in that there is cutaneous involvement. CK levels are
typically high and muscle biopsy would show inflammation within the muscle. Treatment is steroids
initially. A percentage of patients will have an underlying malignancy so it is important that this is
screened for.

Q2. An 80 year old lady develops acute significant pain, stiffness and weakness of her pelvic and
shoulder girdle. She is unable to dress herself or brush her hair. She also complains of a temporal
headache. Her ESR is 90mm/h.

d). Polymyalgia rheumatica (PMR). The age of the patient is a good clue to diagnosis as generally
patients with PMR are elderly. Lack of functional ability is common. Elevated ESR is another
hallmark feature and there is an association with temporal arteritis which is characteristed by a
temporal headache, jaw claudication and in severe cases the optic nerve can be affected. It
responds very dramatically to high dose steroids which should be tapered quickly. If there is no
response to steroids then the diagnosis should be questioned.

Q3. A 50 year old man complains of pain in his long bones and pelvis. On examination he has
proximal muscle weakness and a waddling gait. He has a history of chronic liver disease, but no
history of alcohol abuse. Laboratory tests show a low serum calcium level and an elevated serum
alkaline phosphatase (ALP).

g). Osteomalacia. Osteomalacia occurs as a result of impaired mineralization of bone. It can happen
as a result of Vitamin D deficiency, abnormal metabolism of Vitamin D (eg in liver disease), low
phosphate levels or the presence of bone mineralization inhibitors such as aluminium. It results in
pain, deformity and proximal myopathy. Typically calcium and phosphate levels are low, while
alkaline phosphatase levels are high. If Vitamin D deficiency occurs during the period of bone
growth it is known as rickets.

Q4. A 45 year old lady become unable to rise from a squatting position. She develops acne, truncal
obesity, headaches and very thin skin with purpura. Fasting glucose levels are elevated. A CT scan
shows a pituitary lesion.

h). Cushing’s Disease. A pituitary tumour is secreting ACTH leading to excess cortisol and the typical
features described above. Proximal myopathy can also occur as a result of iatrogenic/therapeutic
steroid use given for respiratory or rheumatological disorders.
www.AceMedicine.com

Q5. A 40 year old patient describes weight loss and palpitations. Her family have commented that
her eyes are very prominent. Examination shows pulse 98 bpm irregularly irregular, proximal
myopathy, sweaty skin, and a palpable neck mass.

c). Thyrotoxicosis. Excess thyroxine hormone causes eye signs, goitre, atrial fibrillation, weight loss,
proximal myopathy. The symptoms of hypothyroidism are different and include cold intolerance,
weight gain, bradycardia, coarse hair and hoarseness.

Rheumatology EMQ 8
Causes of chrondocalcinosis

Which of the following is the most appropriate course of action in each case?

a) Start IV antibiotics immediately then aspirate the joint.


b) Aspiration of joint to look for CPPD crystals, check serum calcium levels.
c) Start oral methotrexate
d) X – ray hands to look for CPPD deposition and ‘hook-like’ spurs at the MCP joints, check serum
ferritin.
e) Aspirate joints and perform synovial fluid microscopy to look for CPPD crystals, x-ray joints and
check anti-CCP antibody.
f) Start intra-muscular Gold injections
g) Aspirate the joint and start IV antibiotics. If no growth on culture inject with steroid.
h) A short course of tapering oral prednisolone followed by prophylactic low dose oral colchicine.
i) Check serum magnesium and serum phosphate
j) Perform full auto-antibody screen

Q1. A 50 year old man has a swollen painful wrist. He also complains of excessive thirst and low
mood. He has a history of renal stones.

b). Aspiration of joint to look for CPPD crystals, check serum calcium levels.

Q2. A 45 year old man is discovered to have abnormal liver function tests. He has bronze
pigmentation of the skin. Fasting glucose levels are elevated and his MCP joints are swollen and
painful.

d). X-ray hands to look for CPPD deposition and ‘hook-like’ spurs at the MCP joints, check serum
ferritin.

The most likely diagnosis is haemachromatosis which is a recognised cause of chondrocalcinosis.


Serum ferritin and transferrin saturation are useful investigations. In addition to the usual
radiological features of CPPD, patients with haemachromatosis can also have degenerative changes
at the MCP joints (‘hook-like’ spurs).
www.AceMedicine.com

Q3. A 50 year old man has swollen knees, elbows and wrists which have evolved over several weeks.
Rheumatoid factor is mildly elevated. He complains also of stiffness and fatigue.

e). Aspirate joints and perform synovial fluid microscopy to look for CPPD crystals, x-ray joints and
check anti-CCP antibody.

Rheumatoid and pseudogout can be confused as their presentations can be similar. The presence of
CPPD crystals will confirm the diagnosis. Anti- CCP antibody is more specific for RA and should be
negative in patients with CPPD even though the RF may be mildly false positive. X-rays should also
help to differentiate between both conditions.

Q4. A 64 year old lady presents with an acutely painful swollen wrist. She is apyrexic, but CRP and
WCC are mildly elevated.

g). Aspirate the joint and start IV antibiotics. If no growth on culture inject with steroid.

It can be difficult to distinguish acute pseudogout from sepsis and if there is any doubt IV antibiotics
should be given until the culture results are known. The presence of CPPD crystals on microscopy
will confirm the diagnosis and a therapeutic steroid injection can be given.

Q5. A 50 year old man suffers from repeated episodes of pseudogout affecting his knees, wrists and
and MCP joints. Acute steroid injections are helpful, but often multiple joints are affected.

f). A short course of tapering oral prednisolone followed by prophylactic low dose oral colchicine.

Colchicine in known to interrupt attacks of pseudogout, however it must be used with care
especially in the elderly as it can cause diarrhoea. Steroids should not be used in the long term, but
can be helpful to settle an acute attack involving multiple joints

Editors: Ian Bickle & Paul Hamilton

Question Writers:

Cardiology by Navtej Chahal & Chris Lockhart


Endocrinology by Fionnula Pollock
Rheumatology by Elisabeth Ball
Data interpretation by Ian Bickle
www.AceMedicine.com

Potrebbero piacerti anche